• Aucun résultat trouvé

REAL NUMBERS – 3

N/A
N/A
Protected

Academic year: 2022

Partager "REAL NUMBERS – 3"

Copied!
24
0
0

Texte intégral

(1)

MAT246H1-S – LEC0201/9201

Concepts in Abstract Mathematics

REAL NUMBERS – 3

(2)

Theorem: ℚ is dense in ℝ

∀𝑥, 𝑦 ∈ ℝ, 𝑥 < 𝑦 ⇒ (∃𝑞 ∈ ℚ, 𝑥 < 𝑞 < 𝑦)

Proof.Let𝑥, 𝑦 ∈ ℝbe such that𝑥 < 𝑦. Set𝜀 = 𝑦 − 𝑥 > 0.

By the archimedean property, there exists𝑛 ∈ ℕ ⧵ {0}such that𝑛𝜀 > 1, i.e. 1𝑛 < 𝜀.

Set𝑚 = ⌊𝑛𝑥⌋ + 1. Then𝑛𝑥 < 𝑚 ≤ 𝑛𝑥 + 1, so𝑥 < 𝑚𝑛 ≤ 𝑥 +1𝑛 < 𝑥 + 𝜀 = 𝑦. ■

Jean-Baptiste Campesato MAT246H1-S – LEC0201/9201 – Mar 18, 2021 2 / 24

(3)

ℚ is dense in ℝ – 2

Definition: interval

A subset𝐼 ⊂ ℝis anintervalif∀𝑥, 𝑦 ∈ 𝐼, ∀𝑧 ∈ ℝ, (𝑥 ≤ 𝑧 ≤ 𝑦 ⇒ 𝑧 ∈ 𝐼).

Crollary

If𝐼 ⊂ ℝis a non-empty interval not reduced to a singleton then𝐼 ∩ ℚ ≠ ∅.

Proof.Since𝐼 is non-empty and not reduced to a singleton, there exist𝑥, 𝑦 ∈ 𝐼with𝑥 < 𝑦.

Then, sinceℚis dense inℝ, there exists𝑞 ∈ ℚsuch that𝑥 < 𝑞 < 𝑦.

Since𝐼is an interval,𝑞 ∈ 𝐼. Hence𝑞 ∈ 𝐼 ∩ ℚ ≠ ∅. ■

(4)

Theorem: ℝ ⧵ ℚ is dense in ℝ

∀𝑥, 𝑦 ∈ ℝ, 𝑥 < 𝑦 ⟹ (∃𝑠 ∈ ℝ ⧵ ℚ, 𝑥 < 𝑠 < 𝑦) Proof.Let𝑥, 𝑦 ∈ ℝsuch that𝑥 < 𝑦.

Since , there exists𝑞 ∈ ℚsuch that𝑥 < 𝑞 < 𝑦.

Similarly, there exists𝑝 ∈ ℚsuch that𝑥 < 𝑝 < 𝑞.

Hence we obtained𝑝, 𝑞 ∈ ℚsuch that𝑥 < 𝑝 < 𝑞 < 𝑦.

Set𝑠 = 𝑝 +√22 (𝑞 − 𝑝). Then𝑠 ∈ ℝ ⧵ ℚ(otherwise, by contradiction,√2would be inℚ) and 𝑝 < 𝑠 < 𝑞(notice that0 <√22 < 1so𝑠is a number between𝑝and𝑞).

We obtained𝑠 ∈ ℝ ⧵ ℚsuch that𝑥 < 𝑠 < 𝑦. ■

I wanted to give it as a question in PS4… before remembering it was in my lecture notes…

Corollary

If𝐼 ⊂ ℝis an interval which is non-empty and not reduced to a singleton then𝐼 ∩ (ℝ ⧵ ℚ) ≠ ∅.

Jean-Baptiste Campesato MAT246H1-S – LEC0201/9201 – Mar 18, 2021 4 / 24

(5)

Decimal representation of real numbers – 1

Lemma

Let(𝑎𝑘)𝑘≥1 be a sequence such that∀𝑘 ∈ ℕ ⧵ {0}, 𝑎𝑘∈ {0, 1, … , 9}. Then the series

𝑆 =

+∞

𝑘=1

𝑎𝑘 10𝑘 is convergent and𝑆 ≥ 0.

Proof.

Note that0 ≤ 𝑎𝑘 10𝑘 ≤ 9

10𝑘 and that

+∞

𝑘=1

9

10𝑘 is convergent (geometric series with ratio 1 10 < 1).

Therefore we may conclude using the BCT. ■

(6)

Unfortunately the decimal representation may not be unique:0.9999 … =

+∞

𝑘=1

9 10𝑘 = 9

10× 1

1 −101 = 1.000 … In order to achieve uniqueness we are going to restrict to expansions which don’t end with infinitely many9.

Definition: proper decimal expansion

Let𝑥 ∈ ℝ. We say that⌊𝑥⌋.𝑎1𝑎2𝑎3… ≔ ⌊𝑥⌋ +

+∞

𝑘=1

𝑎𝑘

10𝑘 is aproper decimal expansion of𝑥if 1 ∀𝑘 ∈ ℕ ⧵ {0}, 𝑎𝑘∈ {0, 1, … , 9} 2 ∀𝑛 ∈ ℕ ⧵ {0},

𝑛

𝑘=1

𝑎𝑘

10𝑘 ≤ 𝑥 − ⌊𝑥⌋ <

𝑛

𝑘=1

𝑎𝑘 10𝑘+ 1

10𝑛

Proposition

If⌊𝑥⌋ +

+∞

𝑘=1

𝑎𝑘

10𝑘is a proper decimal expansion of𝑥 ∈ ℝthen 1 𝑥 = ⌊𝑥⌋ +

+∞

𝑘=1

𝑎𝑘

10𝑘 2 ∀𝑁 ∈ ℕ ⧵ {0}, ∃𝑘 > 𝑁, 𝑎𝑘≠ 9 Proof.

1 We already proved that𝑆 =

+∞

𝑘=1

𝑎𝑘

10𝑘 is convergent. Hence we get𝑆 ≤ 𝑥 − ⌊𝑥⌋ ≤ 𝑆. So𝑥 = ⌊𝑥⌋ + 𝑆.

2 Assume by contradiction that there exists𝑁 ∈ ℕ ⧵ {0}such that∀𝑘 > 𝑁, 𝑎𝑘= 9.

Then𝑥 − ⌊𝑥⌋ =

+∞

𝑘=1

𝑎𝑘 10𝑘=

𝑁

𝑘=1

𝑎𝑘 10𝑘+

+∞

𝑘=𝑁+1 9 10𝑘 =

𝑁

𝑘=1

𝑎𝑘 10𝑘+ 1

10𝑁. Which contradicts the definition of proper decimal expansion.

Jean-Baptiste Campesato MAT246H1-S – LEC0201/9201 – Mar 18, 2021 6 / 24

(7)

Decimal representation of real numbers – 3

Theorem

A real number𝑥admits a unique proper decimal expansion.

Proof.Let𝑥 ∈ ℝ. Up to replacing𝑥with𝑥 − ⌊𝑥⌋, we may assume that⌊𝑥⌋ = 0.

Assume that

+∞

𝑘=1

𝑎𝑘

10𝑘is a proper decimal expansion of𝑥. Then𝑎𝑛≤ 10𝑛 (𝑥 −

𝑛−1

𝑘=1

𝑎𝑘

10𝑘)< 𝑎𝑛+ 1.

So the only possible suitable sequence(𝑎𝑛)is given by𝑎1= ⌊10𝑥⌋and𝑎𝑛+1=

10𝑛+1 (𝑥 −

𝑛

𝑘=1

𝑎𝑘 10𝑘)⌋. It proves the uniqueness, but we still need to check that it is valid.

1 Since⌊𝑥⌋ = 0, we have0 ≤ 𝑥 < 1. Thus0 ≤ 10𝑥 < 10. Therefore𝑎1= ⌊10𝑥⌋ ∈ {0, 1, … , 9}.

Let𝑛 ∈ ℕ ⧵ {0}, then0 ≤ 10𝑛 (𝑥 −

𝑛−1

𝑘=1

𝑎𝑘

10𝑘)− 𝑎𝑛< 1. Thus0 ≤ 10𝑛+1 (𝑥 −

𝑛

𝑘=1

𝑎𝑘 10𝑘)< 10.

Therefore𝑎𝑛+1∈ {0, 1, … , 9}.

2 We have∀𝑛 ∈ ℕ ⧵ {0},

𝑛

𝑘=1

𝑎𝑘 10𝑘≤ 𝑥 <

𝑛

𝑘=1

𝑎𝑘 10𝑘+ 1

10𝑛 by construction.

(8)

Theorem

A real number𝑥 ∈ ℝis rational if and only if its proper decimal expansion is eventually periodic, i.e. ∃𝑟 ∈ ℕ, ∃𝑠 ∈ ℕ ⧵ {0}, ∀𝑘 ∈ ℕ, 𝑎𝑟+𝑘+𝑠= 𝑎𝑟+𝑘, so that

𝑥 = ⌊𝑥⌋.𝑏1𝑏2… 𝑏𝑟𝑐1𝑐2… 𝑐𝑠≔ ⌊𝑥⌋.𝑏1𝑏2… 𝑏𝑟𝑐1𝑐2… 𝑐𝑠𝑐1𝑐2… 𝑐𝑠𝑐1

Proof.

Let𝑥 =𝑎𝑏where𝑎 ∈ ℤand𝑏 ∈ ℕ ⧵ {0}.

By Euclidean division,𝑎 = 𝑏𝑞0+ 𝑟0where0 ≤ 𝑟0< 𝑏. Hence𝑎

𝑏= 𝑞0+𝑟0

𝑏. Note that𝑞0= ⌊𝑎𝑏. And we repeat:10𝑟𝑘= 𝑏𝑞𝑘+1+ 𝑟𝑘+1where0 ≤ 𝑟𝑘+1< 𝑏.

Since there are only𝑏possible remainders, the process will start looping after at most𝑏steps.

But note that the(𝑞𝑘)𝑘≥1defines exactly the decimal expansion of𝑥.

Assume that the proper decimal expansion𝑥 = ⌊𝑥⌋ +

+∞

𝑘=1

𝑎𝑘

10𝑘 is eventually periodic, i.e.∃𝑟 ∈ ℕ, ∃𝑠 ∈ ℕ ⧵ {0}, ∀𝑘 ∈ ℕ, 𝑎𝑟+𝑘+𝑠= 𝑎𝑟+𝑘. Then𝑥 = ⌊𝑥⌋ +

𝑟

𝑘=1

𝑎𝑘 10𝑘+ 10−𝑟

+∞

𝑘=1

𝑎𝑟+𝑘

10𝑘. Hence it is enough to prove that𝑦 =

+∞

𝑘=1

𝑎𝑟+𝑘 10𝑘 ∈ ℚ.

Note that10𝑠𝑦 = 𝑁 + 𝑦where𝑁 = 𝑎𝑟+1𝑎𝑟+2… 𝑎𝑟+𝑠10∈ ℕ. Hence𝑦 =10𝑠 −1𝑁 ∈ ℚ.

Jean-Baptiste Campesato MAT246H1-S – LEC0201/9201 – Mar 18, 2021 8 / 24

(9)

Decimal representation of real numbers – 5

Example

We want to find the decimal expansion of 1529327

24975 .

1 1529327 = 24975 × 61 + 5852

2 58520 = 24975 × 2 + 8570

3 85700 = 24975 × 3 + 10775

4 107750 = 24975 × 4 + 7850

5 78500 = 24975 × 3 + 3575

6 35750 = 24975 × 1 + 10775

And we start to loop. Therefore 152932724975 = 61.234314

(10)

According to the above proof,

𝑎𝑡𝑎𝑡−1… 𝑎0.𝑏1𝑏2… 𝑏𝑟𝑐1𝑐2… 𝑐𝑠= 𝑎𝑡𝑎𝑡−1⋯ 𝑎010+

𝑟

𝑘=1

𝑏𝑘

10𝑘 + 10−𝑟𝑐1𝑐2… 𝑐𝑠10 10𝑠− 1

= 𝑎𝑡𝑎𝑡−1… 𝑎010+ 𝑏1𝑏2… 𝑏𝑟10

10𝑟 + 𝑐1𝑐2… 𝑐𝑠10 10𝑟+𝑠− 10𝑟

= 𝑎𝑡𝑎𝑡−1… 𝑎0𝑏1𝑏2… 𝑏𝑟𝑐1𝑐2… 𝑐𝑠10− 𝑎𝑡𝑎𝑡−1… 𝑎0𝑏1𝑏2… 𝑏𝑟10 10𝑟+𝑠− 10𝑟

Examples

• 61.234314 = 61234314 − 61234

106− 103 = 61173080 999000

• 0.3 = 3 − 0 10 − 1 = 3

9

• 42.012 = 42012 − 42

103− 1 = 41970 999

Jean-Baptiste Campesato MAT246H1-S – LEC0201/9201 – Mar 18, 2021 10 / 24

(11)

√2 ∉ ℚ – 1

Proof 1 (Fundamental Theorem of Arithmetic).

Assume by contradiction that√2 = 𝑎𝑏 ∈ ℚ. Then2𝑏2= 𝑎2.

The prime factorization of the LHS has an odd number of primes (counted with exponents) whereas the RHS has an even number of primes (counted with exponents).

Which is impossible since the prime factorization is unique up to order. ■

(12)

√2 ∉ ℚ – 2

Proof 2 (Euclid’s lemma).

Assume by contradiction that√2 = 𝑎𝑏 ∈ ℚwritten in lowest form.

Then2𝑏2= 𝑎2. Therefore2|𝑎2. By Euclid’s lemma,2|𝑎, so𝑎 = 2𝑘.

Thus2𝑏2= 4𝑘2, from which we get𝑏2= 2𝑘2. By Euclid’s lemma,2|𝑏.

Hence2|gcd(𝑎, 𝑏) = 1, which is a contradiction. ■

Jean-Baptiste Campesato MAT246H1-S – LEC0201/9201 – Mar 18, 2021 12 / 24

(13)

√2 ∉ ℚ – 3

Proof 3 (Gauss’ lemma).

Assume by contradiction that√2 = 𝑎𝑏 ∈ ℚwritten in lowest form.

Then2𝑏2= 𝑎2. Therefore𝑏|𝑎2.

Sincegcd(𝑎, 𝑏) = 1, by Gauss’ lemma (applied twice),𝑏|1and hence𝑏 = 1(since𝑏 ∈ ℕ ⧵ {0}).

Hence𝑎2= 2.

Which is impossible (2is not a perfect square: ∀𝑥 ∈ ℤ, 𝑥2≡ 0 mod3or𝑥2≡ 1 mod3). ■

(14)

√2 ∉ ℚ – 4

Proof 4 (proof by infinite descent).

Assume by contradiction that√2 = 𝑎𝑏 ∈ ℚwhere𝑎 ∈ ℕand𝑏 ∈ ℕ ⧵ {0}.

Then2𝑏2= 𝑎2. Then𝑎(𝑎 − 𝑏) = 𝑎2− 𝑎𝑏 = 2𝑏2− 𝑎𝑏 = 𝑏(2𝑏 − 𝑎). Hence√2 = 𝑎𝑏 = 2𝑏−𝑎𝑎−𝑏. Note that1 < √2 =𝑎𝑏, thus0 < 𝑎 − 𝑏. Therefore0 < 2𝑏 − 𝑎, so𝑎 − 𝑏 < 𝑏.

Therefore we obtained another expression of√2with a smaller positive denominator.

By repeating this process, we may construct an infinite sequence√2 =𝑎𝑏 = 𝑎𝑏1

1 = 𝑎𝑏2

2 = ⋯such

that𝑎𝑘> 0and0 < 𝑏𝑘+1< 𝑏𝑘.

Which is a contradiction since there is no decreasing infinite sequence of natural numbers. ■

Jean-Baptiste Campesato MAT246H1-S – LEC0201/9201 – Mar 18, 2021 14 / 24

(15)

√2 ∉ ℚ – 5

Proof 5 (by congruences).

Assume by contradiction that√2 = 𝑎𝑏 ∈ ℚwritten in lowest form. Then2𝑏2= 𝑎2.

We can’t have𝑎 ≡ 0 mod3and𝑏 ≡ 0 mod3simulatenously (otherwise3|gcd(𝑎, 𝑏) = 1).

• Either𝑎 ≡ ±1 mod3and𝑏 ≡ 0 mod3, then𝑎2− 2𝑏2≡ 1 mod3,

• or𝑎 ≡ 0 mod3and𝑏 ≡ ±1 mod3, then𝑎2− 2𝑏2≡ 1 mod3,

• or𝑎 ≡ ±1 mod3and𝑏 ≡ ±1 mod3, then𝑎2− 2𝑏2≡ 2 mod3.

Therefore𝑎2− 2𝑏2≢ 0 mod3and so𝑎2− 2𝑏2≠ 0. Which is a contradiction. ■

(16)

√2 ∉ ℚ – 6

Proof 6 (by the well-ordering principle).

Assume by contradiction that√2 = 𝑎𝑏 ∈ ℚ. Then𝑎 = √2𝑏.

Therefore𝐸 = {𝑛 ∈ ℕ ∶ 𝑛√2 ∈ ℕ ⧵ {0}}is not empty since it contains|𝑏|as√2|𝑏| = |𝑎|.

By the well-ordering principle,𝐸admits a least element𝑝. Then𝑝√2 ∈ ℕ ⧵ {0}.

Set𝑞 = 𝑝√2 − 𝑝. Then𝑞 ∈ ℤ. Besides𝑞 = 𝑝(√2 − 1)so that0 < 𝑞 < 𝑝.

But𝑞√2 = 2𝑝 − 𝑝√2 = 𝑝 − 𝑞 ∈ ℕ ⧵ {0}. So𝑞 ∈ 𝐸.

Which is a contradiction since𝑝is the least element of𝐸and𝑞 < 𝑝. ■

Jean-Baptiste Campesato MAT246H1-S – LEC0201/9201 – Mar 18, 2021 16 / 24

(17)

√2 ∉ ℚ – 7

Proof 7 (by the rational root theorem).

Assume by contradiction that√2 = 𝑎𝑏 ∈ ℚwritten in lowest form.

Since√2 =𝑎𝑏 is a root of𝑥2− 2 = 0, we deduce from the rational root theorem that𝑎|2and𝑏|1.

So either√2 = ±1or√2 = ±2.

We obtain a contradiction in both cases since(±1)2= 1 ≠ 2and(±2)2= 4 ≠ 2. ■

(18)

√2 ∉ ℚ – 8

Proof 8 (by the archimedean property).

For𝑛 ∈ ℕ, set𝑢𝑛= (√2 − 1)𝑛. We may prove either by induction or using the binomial formula, that for every𝑛, there exist𝑎𝑛, 𝑏𝑛∈ ℤsuch that𝑢𝑛= 𝑎𝑛+ 𝑏𝑛√2.

Since10 < √2 − 1 < 12, we may also prove that0 < 𝑢𝑛21𝑛. Assume by contradiction that√2 = 𝑝𝑞 ∈ ℚ, then

𝑢𝑛= 𝑎𝑛+ 𝑏𝑛√2 = 𝑎𝑛+ 𝑏𝑛𝑝

𝑞 = 𝑞𝑎𝑛+ 𝑝𝑏𝑛 𝑞 Since𝑢𝑛> 0we get that|𝑞𝑎𝑛+ 𝑝𝑏𝑛| ≥ 1and that𝑢𝑛|𝑞|1.

Therefore∀𝑛 ∈ ℕ, 0 < |𝑞|1 ≤ 𝑢𝑛21𝑛. Which contradicts the archimedean property. ■

1Use the fact that(0, +∞) ∋ 𝑥 → 𝑥2∈ ℝis increasing and that2 ≤ (32)

2

to conclude that√2 ≤32.

Jean-Baptiste Campesato MAT246H1-S – LEC0201/9201 – Mar 18, 2021 18 / 24

(19)

√2 ∉ ℚ – 9

Proof 9 (geometric version of proof 4).

Assume by contradiction that√2 = 𝑎

𝑏 ∈ ℚwhere𝑎 ∈ ℕand𝑏 ∈ ℕ ⧵ {0}. Then𝑎 = √2𝑏 > 𝑏.

𝑎 𝑏

𝑏 𝑎

𝑏 𝑏

(𝑎 − 𝑏)2

(𝑎 − 𝑏)2 (2𝑏 − 𝑎)2

By a direct computation of the side length, the square at the center has an area of𝒜 = (2𝑏 − 𝑎)2.

But, by inclusion-exclusion,𝒜also satisfies2(𝑎 − 𝑏)2+ 2𝑏2− 𝒜 = 𝑎2. So𝒜 = 2(𝑎 − 𝑏)2+ 2𝑏2− 𝑎2= 2(𝑎 − 𝑏)2since𝑎2= 2𝑏2.

Therefore2(𝑎 − 𝑏)2= 𝒜 = (2𝑏 − 𝑎)2. Thus2 =(2𝑏−𝑎)(𝑎−𝑏)22. Hence√2 = 2𝑏−𝑎

𝑎−𝑏 witha0 < 2𝑏 − 𝑎and0 < 𝑎 − 𝑏 < 𝑏.

aSee Proof 4 for0 < 2𝑏 − 𝑎.

By repeating this process, we may construct an infinite sequence√2 =𝑎

𝑏 = 𝑎1

𝑏1 =𝑎2

𝑏2 = ⋯such that𝑎𝑘> 0 and0 < 𝑏𝑘+1< 𝑏𝑘.

Which is a contradiction since there is no decreasing infinite sequence of natural numbers. ■

(20)

√2 ∉ ℚ – 10

Proof 10 (Pythagoras flavored).

Let𝐴𝐵𝐶be a isosceles right triangle in𝐴. By the Pythagorean theorem𝐵𝐶

𝐴𝐵= √2.

Assuming that√2is rational means geometrically that𝐵𝐶and𝐴𝐵are commensurable, i.e. they are both integral multiple of a another length𝑑2.

Put𝐷on[𝐵𝐶]such that𝐵𝐷 = 𝐴𝐵.

Define𝐸as the intersection of(𝐴𝐶)with the line through𝐷which is perpendicular to(𝐵𝐶).

Note that3𝐴𝐸 = 𝐸𝐷 = 𝐷𝐶.

Thus𝐶𝐷 = 𝐵𝐶 − 𝐴𝐵and𝐸𝐶 = 𝐴𝐶 − 𝐴𝐸 = 𝐴𝐵 − (𝐵𝐶 − 𝐴𝐵) = 2𝐴𝐵 − 𝐵𝐶.

Therefore𝐶𝐷and𝐸𝐶are integral multiple of𝑑.

Besides𝐷𝐸𝐶is a isosceles right triangle in𝐷, therefore we may repeat this construction on the triangle𝐷𝐸𝐶in order to construct an infinite sequence of segment lines (𝐴𝐶, 𝐸𝐶, 𝐹 𝐶, …, see below) which are all integral multiple of𝑑and with decreasing length.

Which is impossible.

𝐴 𝐵

𝐶

𝐴 𝐵

𝐶 𝐷 𝐸

𝐴 𝐵

𝐶 𝐷 𝐸

𝐹 𝐺

2That is the geometric version ofirrationalityused by ancient Greeks:

if√2 = 𝑎𝑏, set𝑑 = 𝐴𝐵𝑏 then𝐴𝐵 = 𝑏𝑑and𝐵𝐶 = √2 × 𝐴𝐵 = 𝑎𝑏𝑏𝑑 = 𝑎𝑑.

3Compare the triangles𝐵𝐴𝐸and𝐵𝐷𝐸which are respectively right in𝐴and𝐷with common hypotenuse and𝐴𝐵 = 𝐷𝐵, so, by the Pythagorean theorem,𝐴𝐸 = 𝐸𝐷.

Besides the triangle𝐶𝐷𝐸is isosceles right in𝐴by angle considerations, thus𝐸𝐷 = 𝐷𝐶.

Jean-Baptiste Campesato MAT246H1-S – LEC0201/9201 – Mar 18, 2021 20 / 24

(21)

√2 ∉ ℚ – 11

Proof 11 (my favorite one).

The proof is left as an exercise to the reader. ■

(22)

More about √2

Before leaving√2, I would like to show you a funny proof relying on thetertium non datur.

Proposition

There exist𝑎, 𝑏 > 0irrational numbers such that𝑎𝑏∈ ℚ.

Proof.

• Assume that√2√2∈ ℚ. Then we can take𝑎 = 𝑏 = √2.

• Assume that√2√2∉ ℚ. Then we can take𝑎 = √2√2and𝑏 = √2.

Indeed,

(√2√2 )

√2

= √22= 2.

Jean-Baptiste Campesato MAT246H1-S – LEC0201/9201 – Mar 18, 2021 22 / 24

(23)

𝑒 ∉ ℚ

Recall from your first year calculus that𝑒 =

+∞

𝑛=0

1 𝑛!. Proof 1.

Assume by contradiction that𝑒 = 𝑎𝑏 where𝑎, 𝑏 ∈ ℕ ⧵ {0}. Note that𝑏 > 1since𝑒 ∉ ℕ. Besides

𝑏!

⎛⎜

⎜⎝ 𝑒 −

𝑏

𝑛=0

1 𝑛!

⎞⎟

⎟⎠

= 𝑏!( ∑𝑛≥𝑏+1 1 𝑛! ) Note that the LHS is an integer.

We are going to derive a contradiction by proving that the RHS is not an integer. Indeed 0 < 𝑏!

( ∑𝑛≥𝑏+1 1

𝑛! )≤∑

𝑛≥1

1 (𝑏 + 1)𝑛 = 1

𝑏 < 1

(24)

Proof 2.

For𝑛 ∈ ℕ, set𝑢𝑛=

1 0

𝑥𝑛𝑒𝑥d𝑥.

Using an induction and integration by part, we can prove that for𝑛 ∈ ℕ, there exist𝑎𝑛, 𝑏𝑛∈ ℤ such that𝑢𝑛= 𝑎𝑛+ 𝑒𝑏𝑛.

Assume by contradiction that𝑒 = 𝑝𝑞 where𝑝, 𝑞 ∈ ℕ ⧵ {0}. Then0 < 𝑢𝑛= 𝑎𝑛+ 𝑏𝑛𝑝𝑞 = 𝑞𝑎𝑛+𝑝𝑏𝑞 𝑛. Since𝑢𝑛> 0we get that𝑞𝑎𝑛+ 𝑝𝑏𝑛≥ 1and that𝑢𝑛1𝑞.

Therefore∀𝑛 ∈ ℕ, 0 < 1 𝑞 ≤ 𝑢𝑛

1 0

𝑥𝑛𝑒d𝑥 = 𝑒 𝑛 + 1.

Which is impossible. ■

Jean-Baptiste Campesato MAT246H1-S – LEC0201/9201 – Mar 18, 2021 24 / 24

Références

Documents relatifs

As well, compressed air foam can be used with Class A foam concentrate, which is environmentally friend- ly, rather than Class B foam concentrate, which is used in traditional

(For the definition of the diophantine notions used from now on, see Section 1.3.) One of our results will be that the sequence (G 1,N (α)) N≥1 converges for all irrational numbers

First we prove that Cauchy reals setoid equality is decidable on algebraic Cauchy reals, then we build arithmetic operations.. 3.2 Equality

Sollte eine MRT durchgeführt werden, kann es durch die magnetostatische Magnetfeldstärke von bis zu 3 T, die mittlerweile postmortal Anwendung findet, zu einer Änderung in

rural products, value addition at producers' level and the improvement of value chain performance through market systems development; enhancing the capacity of rural producers'

We address the problem of the maximal finite number of real points of a real algebraic curve (of a given degree and, sometimes, genus) in the projective plane.. We improve the

When taking into account the size of the RJV, we find that the positive impact on labor productivity comes mainly from participation in large projects and that participation in

The model is calibrated using historical data on England concerning both economic growth rate and the factor shares (land, capital, and labor) in total income, as well as